5
$\begingroup$

Let $G$ be a finite group. Let $M$ be a finite $G$-module (a finite abelian group with an action of $G$). We consider a special kind of $G$-modules; in particular, our $M$ is a finite dimensional representation of $G$ over $\mathbb{F}_p$. For our $G$-modules $M$, we ask whether it is possible that $Ш^1_\omega(G,M)\ne 0$.

We ask our question with a hope to answer this hard question (in positive or negative). The relation is the following. Let $L/k$ be a Galois extension of number fields with Galois group $G$. Then we may regard $M$ as a $\mathrm{Gal}(\bar{k}/k)$-module. In this case $$ Ш^1(k,M)\subset Ш^1_\omega(G,M).$$ Moreover, if the decomposition groups of all the places of $k$ in $\mathrm{Gal}(L/k)=G$ are cyclic, then $$ Ш^1(k,M)= Ш^1_\omega(G,M).$$ Here $ Ш^1(k,M)$ is the "honest" Tate-Shafarevich group of the $\mathrm{Gal}(\bar{k}/k)$-module $M$.

Following Sansuc, we define $$Ш^1_\omega(G,M)=\mathrm{ker}\left[H^1(G,M)\to\prod_C H^1(C,M)\right],$$ where $C$ runs over the set of cyclic subgroups of $G$. We write $Ш(G,M)$ for $Ш^1_\omega(G,M)$. If $G$ acts trivially on $M$, then clearly $Ш(G,M)=0$ (because then $H^1(G,M)=\mathrm{Hom}(G,M)$ ). Sansuc proves that if all the Sylow subgroups of $G$ are cyclic, then $Ш(G,M)=0$ for any $G$-module $M$. Using his method, we prove the following proposition:

Proposition. Let $p$ be a prime number. If $M$ is a $G$-module such that $p^n M=0$ for some $n\ge 1$, and if a Sylow $p$-subgroup of $G$ is cyclic, then $Ш(G,M)=0$.

Proof. Let $P$ be a Sylow $p$-subgroup of $G$, then the map $$ \mathrm{Cor}\circ\mathrm{Res}\colon H^1(G,M)\to H^1(P,M)\to H^1(G,M)$$ is the multiplication by $[G:P]$. Since $P$ is cyclic, this map is 0 on $Ш(G,M)$. Since $p^n M=0$, the multiplication by $p^n$ on $Ш(G,M)$ is 0 as well. Since the numbers $p^n$ and $[G:P]$ are coprime, we conclude that $Ш(G,M)=0$.

Let $H$ be a subgroup of $G$ (e.g., $H=\{1\}$). We consider the $G$-set $X:=G/H$. We embed $\mathbb{F}_p$ into $\mathrm{Maps}(X,\mathbb{F}_p)$ as the subspace of constant maps, and we set $$M(G,H,p):=\mathrm{Maps}(X,\mathbb{F}_p)/\mathbb{F}_p.$$ Then $M(G,H,p)$ is a finite dimensional representation of $G$ over $\mathbb{F}_p$, hence a $G$-module. (For $p=2$, this is the Galois module $T[2]$ from this answer).

Question. Do there exist $G$, $H$, and $p$, such that for $M=M(G,H,p)$ we have $Ш(G,M)\ne 0$?

I would be especially interested in a counter-example with $p=2$.

The proposition above shows that for a counter-example $M(G,H,p)$, the group $G$ must have a noncyclic Sylow $p$-subgroup.

Proposition. If $p\nmid [G:H]$, then for $M=M(G,H,p)$ we have $Ш(G,M)=0$.

Proof (due to user nfdc23). Write $X=G/H$, then $\# X$ is prime to $p$. It follows that $M=M(G,H,p)$ is isomorphic (as a $G$-module) to a direct summand of the $G$-module $\mathrm{Maps}(X,\mathbb{F}_p)$. Since $Ш(G,\mathrm{Maps}(X,\mathbb{F}_p))=0$, we conclude that $Ш(G,M)=0$.

$\endgroup$

1 Answer 1

6
$\begingroup$

I think the following is an example of $Ш(G,M(G,H,\Bbb{F}_2))\neq 0$: Take $G=A_4$ and $H$ of order $2$. Then $M$ has dimension $5$ and a (computer) calculation shows that $Ш(G,M(G,H,\Bbb{F}_2))$ has dimension $1$.

The following is a sketch of how to do the computation by hand. First of all $A_4$ has two conjugacy classes of cyclic subgroups namely $H=\langle (1,2)(3,4)\rangle$ and $K=\langle (1,2,3)\rangle$. Since $\lvert K\rvert=3$ it suffices to prove that the restriction map $H^1(G,M)\rightarrow H^1(H,M)$ is not injective. Let $V_4=\langle (1,2)(3,4),(1,3)(2,4)\rangle$. The spectral sequence associated the sequence $1\rightarrow V_4\rightarrow A_4\rightarrow A_4/V_4\cong C_3\rightarrow 1$ collapses to give $H^*(A_4,M)\cong H^*(V_4,M)^{C_3}$. From this it is a straightforward (but tedious) computation to see that both $H^1(G,M)$ and $H^1(H,M)$ have dimension $1$ and that the restriction map is the zero map.

Here is the Magma code I used to verify the example:

G:=Alt(4); H:=sub<G|(1,2)(3,4)>; M:=PermutationModule(G,H,GF(2)); M:=M/Fix(M); XG:=CohomologyModule(G,M); H1G:=CohomologyGroup(XG,1); XH:=Restriction(XG,H); H1H:=CohomologyGroup(XH,1); ims:=[IdentifyOneCocycle(XH,OneCocycle(XG,H1G.i)) : i in [1..Dimension(H1G)]]; res:=hom<H1G->H1H|ims>; print Dimension(H1G),Dimension(H1H),Dimension(Kernel(res));

The output 1, 1, 1 confirms the computation above.

$\endgroup$
6
  • $\begingroup$ How do you calculate this with a computer? $\endgroup$
    – user19475
    Jun 13, 2016 at 7:38
  • 2
    $\begingroup$ I used Magma, but the example is small enough that one can do it by hand. A reference for the relevant functions in Magma is magma.maths.usyd.edu.au/magma/handbook/… It is also possible to use GAP, see gap-system.org/Manuals/doc/ref/chap39.html#X7CA0B6A27E0BE6B8 $\endgroup$ Jun 13, 2016 at 13:10
  • $\begingroup$ Could you please explain, how one can do it hand? The group $A_4$ is of order 12, and $M$ is of dimension 5, so maybe brute force is not sufficient.... What tricks can I use? Please kindly add this to your answer (you can edit your answer). $\endgroup$ Jun 13, 2016 at 18:08
  • 2
    $\begingroup$ Also, could you please add your Magma code to your answer? I have never used computer algebra, but I am a former programmer, and I would be happy to learn using Magma by this example, and then to compute other examples! $\endgroup$ Jun 13, 2016 at 18:15
  • 1
    $\begingroup$ Dear Mikhail! I added a sketch of how to do the computation by hand and the Magma code I used. Let me know if I need to add more detail. $\endgroup$ Jun 14, 2016 at 19:22

Your Answer

By clicking “Post Your Answer”, you agree to our terms of service and acknowledge you have read our privacy policy.

Not the answer you're looking for? Browse other questions tagged or ask your own question.